2
$\begingroup$

Exercise. A forest is in flames and the government is planning a firefighter operation. The fire is of small dimensions and is progressing slowly and it must be extinct after $3$ hours of operations. There will be helicopters and planes mobilized to the forest. They obey the following table:

\begin{array}{|c|c|c|} \hline \text{Device} & \text{Efficiency $(m^2/hour)$} & \text{Cost (Euros/hour)} & \text{Personal Necessity}\\ \hline \text{Helicopter AH1} & 15000 & 2000 & 2 \text{ pilots}\\ \hline \text{Tank Airplane} & 40000 & 4000 & \text{2 pilots + 1 operator}\\ \hline \text{B67 Airplane} & 85000 & 10000 & \text{2 pilots + 3 operators}\\ \end{array} The area of the forest covered by fire is $3 000 000$ $m^2$. In the support areas, we have $14$ Airplane pilots available, $10$ Helicopter pilots available and $22$ operators available. Model the problem of minimization of the costs of this operation.

My attempt and doubts. I defined the following: \begin{equation*} x_1 \rightarrow \text{ number of activity hours of AH1} \\ x_2 \rightarrow \text{ number of activity hours of Tank} \\ x_3 \rightarrow \text{ number of activity hours of B67} \end{equation*} And I defined the following restrictions: \begin{equation*} 1500x_1 +40000x_2 + 85000x_3 \leq 3000000 \\[.25cm] 0 \leq x_{1,2,3} \leq 3 \end{equation*} The first one is related to the forest area covered by the fire and the second one is related to the hours the operation must hold. I think these are the only restrictions I should take related to these variables. I defined the objective function as the follwing: \begin{equation*} z(x_1,x_2,x_3) = 2000x_1 + 4000x_2 + 10000x_3 \end{equation*} And the objective would be to minimize this function. Now come my problems:

I wasn't able to define restrictions realted to the number of pilots/operators needed. I have thought about adding new variables to do so, but been unable to (somehow everytime I try it doesn't make sense to me). So this is basically why I am posting this, to know how to model the pilots/operators restrictions. Thanks for all the help in advance.

$\endgroup$

1 Answer 1

1
$\begingroup$

In my view we have to fix the operation time at $3$ hours in this model. I don´t see any other way. Then I define the variables in a different way:

\begin{equation*} x_1 \rightarrow \text{ number Helicopters AH1} \\ x_2 \rightarrow \text{ number of Tank Airplanes} \\ x_3 \rightarrow \text{ number of B67 Airplanes} \end{equation*}

Then in 3 hours each of the 3 different aircraft types can cover the following areas:

\begin{equation*} \text{ Helicopter AH1: 15000*3=45000} \\ \text{ Tank Airplane: 40000*3=120000} \\ \text{ B67 Airplane: 85000*3=255000} \end{equation*}

The the area-restriction is

$$45000\cdot x_1+120000\cdot x_2+255000\cdot x_3=3000000$$

The cost per hour has to be multiplied by $3$ as well to obtain the cost per aircraft.

And the restriction for the airplane pilots would be $2x_2+2x_3\leq 14$, for instance. Btw, defining variables is the best what you can do in linear programming. Good job, especially in this regard.

$\endgroup$
2
  • 1
    $\begingroup$ Hi @callculus42 . Sorry for the delay, I have solved the problem using your solution. Thank you for the help, made a lot of sense to me fixing the $3$ hours! I guess that's what I was missing :D $\endgroup$ Commented Nov 12, 2021 at 18:25
  • 1
    $\begingroup$ @Seabourn I can comprehend your confusion. In my view the exsercise isn´t very clear formulated.. $\endgroup$ Commented Nov 12, 2021 at 18:32

You must log in to answer this question.

Start asking to get answers

Find the answer to your question by asking.

Ask question

Explore related questions

See similar questions with these tags.